(frazioni)^3

Numeri interi, razionali, divisibilità, equazioni diofantee, ...
Rispondi
Simo_the_wolf
Moderatore
Messaggi: 1053
Iscritto il: 01 gen 1970, 01:00
Località: Pescara

(frazioni)^3

Messaggio da Simo_the_wolf »

Dimostrare che per ogni numero $ q \in \mathbb{Q}^+ $ esistono interi positivi $ a,b,c,d $ tali che:

$ \displaystyle q=\frac {a^3+b^3}{c^3+d^3} $
Avatar utente
HumanTorch
Messaggi: 281
Iscritto il: 01 gen 1970, 01:00
Località: Tricase

Messaggio da HumanTorch »

Lavorare sui fattori mi sembra calcoloso...induzione?
Perhaps the world is not made.
Perhaps nothing is made.
Perhaps it simply is, has been, will always be there ...
..a clock without craftsman
Who watches the watchmen?

Alan Moore, "Watchmen"
Simo_the_wolf
Moderatore
Messaggi: 1053
Iscritto il: 01 gen 1970, 01:00
Località: Pescara

Messaggio da Simo_the_wolf »

Induzione??? mmm, non saprei....
Avatar utente
Sisifo
Messaggi: 604
Iscritto il: 01 gen 1970, 01:00
Località: Scorzè (VE)/Pisa

Messaggio da Sisifo »

Provo a riscrivere il problema:
Dati l, m interi primi fra loro esiste un h intero tale che sia hl che hm siano esprimibili come somma di due cubi... sono fuori strada?
Avatar utente
info
Messaggi: 903
Iscritto il: 01 gen 1970, 01:00

Messaggio da info »

Dopo avere provato un pò con l'induzione ed essere arrivato ad un'altra diofantea che non so cmq risolvere....può essere utile porre [p/q è il numero razionale]...

a=kp+t
b=fp-t

e analogamente c e d, sostituire, vedere l'equazione ottenuta come una equzione di secondo grado ed utilizzare più volte questo risultato carino che spero abbia una qualche ragione di esistere:

------------
presa l'equazione

ax^2-bx+c=0

con a,b,c appartenenti ad No a,b>0, essa ammette almeno una sol intera positiva sse b=a+c...
-----------

???????????????????????????

ps: il caso q=1 è famoso e dà come risultato minimo per la somma dei cubi al numeratore 1729, come Ramanujan insegna, sempre che le a,b sia diverso da c,d...
Ultima modifica di info il 03 giu 2005, 20:52, modificato 1 volta in totale.
Avatar utente
Pixel
Messaggi: 79
Iscritto il: 23 feb 2005, 16:16
Località: Trento

Messaggio da Pixel »

ponendo q=$ \frac{m}{n} $ e $ a=m-k_1n $, $ b=m+k_1n $ $ c=n-k_2m $ e $ d=n+k_2m $ salta fuori qualcosa di carino, provate un pò.
P. Andrea
Avatar utente
info
Messaggi: 903
Iscritto il: 01 gen 1970, 01:00

Messaggio da info »

sicuro che venga Pixel? Ho provato a sostituire e mi viene che se vale l'eguaglianza deve valere per una qualche coppia di positivi (k1,k2) questa diofantea:

m^2+3k1*n^2=n^2+3k2*m^2

prendendola modulo 3:

m^2=n^2

ovverosia basta prendere m ed n di modo che non sia verificata la divisibilità modulo 3 per fare in modo che la diofantea sopra non abbia soluzioni... Cosa sbaglio o non capisco????
Simo_the_wolf
Moderatore
Messaggi: 1053
Iscritto il: 01 gen 1970, 01:00
Località: Pescara

Messaggio da Simo_the_wolf »

Oppure vuol dire che $ (3k_1-1)m^2=(3k_2-1)n^2 $ quindi basterebbe porre $ 3k_1=n^2+1 $ e $ 3k_2=m^2+1 $. Ma non regge perchè $ n^2+1 $ non è mai multiplo di 3... Però forse con qualche modifica... :D
Avatar utente
info
Messaggi: 903
Iscritto il: 01 gen 1970, 01:00

Messaggio da info »

Forse ci sono riuscito; partendo come prima (il numero razionale è =p/q)

a=kp+t
b=fp-t

c=rq+m
d=sq-m

si eseguono i calcoli. Per sbarazzarsi di un po’ di roba si pone k=f ed r=s, sperano di lasciare qualche sol ed ottenendo la diofantea:

k^3p^2+3kt^2=r^3q^2+3rm^2

pongo inoltre r=k

t^2=[k^2(q^2-p^2)+3m^2]/3

sarebbe stato comodo un k=3c

t^2=3c^2(q^2-p^2)+m^2

t^2-m^2=3c^2(q^2-p^2)

a destra abbiamo un numero dispari pur di scegliere c dispari, a sinistra una diff tra 2 quadrati: possiamo scegliere t, m e c di modo che la diofantea resti soddisfatta.. si può fare anche se q e p sono entrambi dispari scegliendo due quadrati che differiscano di 2 unità... credi che funzioni Simo???? Perché sinceramente c’è un passaggio sul quale non ho tanto le idee chiare :? :? :? :? :cry:
Ultima modifica di info il 05 giu 2005, 20:13, modificato 3 volte in totale.
Avatar utente
Pixel
Messaggi: 79
Iscritto il: 23 feb 2005, 16:16
Località: Trento

Messaggio da Pixel »

Cmq sia info ricorda che a,b,c e d devono anche essere positivi.
P. Andrea
Avatar utente
info
Messaggi: 903
Iscritto il: 01 gen 1970, 01:00

Messaggio da info »

Già è vero: dimenticavo di avere letto questa condizione a dire il vero :shock: ... beh... immagino che tu abbia capito che la sol sopra non si può "salvare" e non controllo... Non ho voglia di provare ancora a dire il vero ma credo che la sostituzione di partenza sia buona...migliori non me ne vengono in mente...

aggiunta:
----------------------
cmq ho controllato ed effettivamente il metodo sopra porta ad infinte sol per ogni numero razionale se allarghiamo il discorso anche ai numeri relativi...
Simo_the_wolf
Moderatore
Messaggi: 1053
Iscritto il: 01 gen 1970, 01:00
Località: Pescara

Messaggio da Simo_the_wolf »

ok, prova a dare a t, m,c valori per cui la diofantea funzioni e poi sostituisci. Per la positività di a,b,c,d vedi le condizioni che dovrebbero sussistere e poi...
Avatar utente
info
Messaggi: 903
Iscritto il: 01 gen 1970, 01:00

Messaggio da info »

mmm... mi pareva di avere provato ieri con qualche esempio numerico e non mi ritrovavo... beh! Ma se dici di provare, mi sarà sfuggito qualcosa!... quando ho tempo e calma sufficiente faccio il tutto per bene... devo dire che già trovare sol per i relativi mi ha dato un pò di soddisfazione :D
Avatar utente
HiTLeuLeR
Messaggi: 1874
Iscritto il: 01 gen 1970, 01:00
Località: Reggio di Calabria

è stata dura, ma alla fine... =)))

Messaggio da HiTLeuLeR »

Se $ q\in\mathbb{Q}^+ $, esistono $ m, n\in\mathbb{N}_0 $ tali che $ q = \dfrac{m}{n} $. Senz'essere lesivi di generalità, possiamo suppore per il seguito $ m > n $. Se $ m < 2n $, si cercano $ a, b, c, d \in \mathbb{N}_0 $ tali che $ \dfrac{m}{n} = \dfrac{a^3 + b^3}{b^3+d^3} = \dfrac{(a+b)(a^2 - ab + b^2)}{(c+d)(c^2 - cd + d^2)} $. Poniamo in tal senso $ b := 2m - n $, $ d := 2n-m $ ed $ a = c := b+d $. Con queste assunzioni, essendo $ \dfrac{1}{2} < \dfrac{m}{n} < 2 $: $ \min(a,b,c,d) > 0 $; $ a+b = 3m $; $ c+d = 3n $ e $ a^2 - ab + b^2 = c^2 - cd + d^2 $, sicché la tesi è prontamente soddisfatta. Se poi $ m \geq 2n $, si cerchi innanzitutto $ t\in\mathbb{Q}^+ $ tale che $ \dfrac{1}{2} < \dfrac{mt^3}{n} < 2 $, condizione certamente possibile per via della densità di $ \mathbb{Q} $ in $ \mathbb{R} $. Posto allora t = x/y, con
$ x, y \in\mathbb{N}_0 $; m' := m x^3 ed n' := n y^3, risulta $ \dfrac{1}{2} < \dfrac{m'}{n'} < 2 $, perciocché - sulla base del precedente stabilito - sono certamente determinati $ \alpha, \beta, \gamma, \delta \in \mathbb{N}_0 $ tali che: $ \dfrac{mx^3}{ny^3} = \dfrac{m'}{n'} = \dfrac{\alpha^3 + \beta^3}{\gamma^3+\delta^3} $. Ne consegue finalmente ch'esistono $ a, b, c, d\in\mathbb{N}_0 $, con $ a := \alpha y $, $ b := \beta y $, $ c := \gamma x $ e $ d := \delta x $, tali che $ \dfrac{a^3 + b^3}{c^3 + d^3} = \dfrac{y^3(\alpha^3 + \beta^3)}{x^3(\gamma^3+\delta^3)} = \dfrac{m}{n} $. Di qui la tesi, q.e.d.
Avatar utente
HiTLeuLeR
Messaggi: 1874
Iscritto il: 01 gen 1970, 01:00
Località: Reggio di Calabria

Messaggio da HiTLeuLeR »

Nella versione precedente della mia soluzione, oltre a un certo numero di typos, ci stava un errore tanto grave che mi meraviglia assai come *nessuno* l'abbia fatto notare!!! :shock: Meglio così, ché altrimenti chi vi stava poi a sentire, uh? :mrgreen:
Rispondi